1. Trang chủ
  2. » Giáo Dục - Đào Tạo

Các bất đẳng thức ag lồi, gg lồi, ga lồi và ứng dụng

32 16 0

Đang tải... (xem toàn văn)

Tài liệu hạn chế xem trước, để xem đầy đủ mời bạn chọn Tải xuống

THÔNG TIN TÀI LIỆU

Nội dung

Mục lục trang Mở đầu Chương Bất đẳng thức lồi 1.1 Hàm lồi không gian Rn 1.1.1 Định nghĩa 1.1.2 Một số tính chất chung 1.1.3 Ví dụ 11 1.2 Hàm lồi biến 16 1.2.1 Đặt vấn đề 16 1.2.2 Các tính chất hàm lồi biến 16 Chương Bất đẳng thức số học hình học 19 2.1 Định nghĩa 19 2.1.1 Hàm AG-lồi 19 2.1.2 Hàm GG-lồi 19 2.1.3 Hàm GA-lồi 19 2.2 Tính chất 20 2.3 Một số áp dụng 22 2.4 Một số ví dụ 25 2.4.1 Bài toán 25 2.4.2 Bài toán 26 2.4.3 Bài toán 28 2.4.4 Bài toán 29 2.4.5 Bài toán 30 2.4.6 Bài toán 31 Kết luận 33 Tài liệu tham khảo 34 Mở đầu Trong lý thuyết Giải tích lồi, biết đến hàm lồi, với bất đẳng thức Jensen có ảnh hưởng lớn đến lý thuyết Bất đẳng thức Trong năm gần đây, nhiều nhà toán học quan tâm tới việc mở rộng số bất đẳng thức khác, thông qua việc nghiên cứu hàm lồi số học (Arithmetic convex functiuon), hàm lồi hình học (Geometric convex functiuon) Các tác giả C.P Niculescu, R.A Satnoianu (xem [5], [6]) nhận nhiều kết thú vị xem xét hàm lồi Xuất phát từ kết mà số tác giả tìm được, tơi thấy cần hiểu biết thêm muốn tiếp cận Vì lý đó, tơi lựa chọn đề tài Các bất đẳng thức AG-lồi, GG-lồi, GA-lồi ứng dụng Nội dung luận văn bao gồm hai chương: Chương 1: Bất đẳng thức lồi Trong chương này, chúng tơi trình bày kiến thức sở có liên quan đến bất đẳng thức lồi, là: Khái niệm hàm lồi khơng gian Rn , Bất đẳng thức Jensen, Hàm lồi biến số số ứng dụng Chương 2: Bất đẳng thức số học hình học Đây nội dung khố luận Trong chương này, sau trình bày bất đẳng thức số học hình học, chúng tơi cố gắng xem xét số tính chất áp dụng Cuối toán áp dụng nhằm minh hoạ cho ảnh hường bất đẳng thức nêu Các kết chương 2, chủ yếu dựa theo kết báo [5], [6], có tham khảo thêm [1] Khố luận thực hồn thành hướng dẫn khoa học PGS TS Trần Xuân Sinh Tác giả xin bày tỏ lòng biết ơn sâu sắc tới thầy hướng dẫn tận tâm thầy tác giả suốt thời gian học tập nghiên cứu Nhân dịp này, tác giả xin gửi lời cảm ơn tới PGS TS Nguyễn Văn Quảng, thầy cô giáo tổ Xác suất thống kê, khoa Toán, khoa Sau Đại học Đồng thời, tác giả xin bày tỏ lòng biết ơn tới gia đình bạn bè, quan tâm, góp ý tạo điều kiện giúp đỡ tác giả thực khoá luận Mặc dù cố gắng song khố luận khơng thể tránh khỏi sai sót Tác giả mong nhận đóng góp quý thầy giáo bạn để khố luận hồn thiện Tác giả xin chân thành cảm ơn! Vinh, tháng 05 năm 2009 Tác giả Chương Bất đẳng thức lồi 1.1 Hàm lồi không gian Rn 1.1.1 Định nghĩa • Hàm f (k) xác định tập lồi M gọi hàm lồi, với x, y ∈ M α ∈ [0, 1] có bất đẳng thức f [αx + (1 − α)y] ≤ αf (x) + (1 − α)f (y) (1) Ta gọi bất đẳng thức (1) bất đẳng thức lồi • Nếu f [αx + (1 − α)y] ≥ αf (x) + (1 − α)f (y) ta nói f hàm lõm Ví dụ: Hàm tuyến tính tuyến tính afin hàm vừa lõm, vừa lồi Hàm y = max{x, a}, với a > hàm lồi (tuyến tính khúc với điểm gẫy x = a) Cho f (x) hàm lồi, liên tục tập lồi M Khi hàm y = max{f (x), 0}, với x ∈ M hàm lồi 1.1.2 Một số tính chất chung • Bất đẳng thức Jensen Định lý Cho tập lồi M ⊂ R, xi ∈ M, i = 1, k, λi ≥ 0, k i=1 λi = Hàm f hàm lồi tập lồi M k k λi x i ≤ f i=1 λi f (xi ) i=1 (2) Chứng minh Giả sử có bất đẳng thức (2), với k = 2, theo định nghĩa f hàm lồi Ngược lại, giả sử f (x) hàm lồi M Xét k k λi xi , ∀λi ≥ 0, x= i=1 λi = 1, xi ∈ M i=1 Rõ ràng M lồi nên x ∈ M Để chứng minh (2) ta quy nạp theo k Với k = 2, bất đẳng thức Giả sử bất đẳng thức với k − 1, ta chứng minh với k Ta có k k−1 λi f (xi ) = i=1 λi f (xi ) + λk f (xk ) i=1 Khơng tính tổng quát, giả sử < λk < Khi k k−1 λi f (xi ) = (1 − λk ) i=1 i=1 λi f (xi ) + λk f (xk ) − λk k−1 = λk f (xk ) + (1 − λk ) βi f (xi ) i=1 với βi = λi 1−λk ≥ 0, ∀i = 1, 2, , k − 1, rõ ràng k−1 i=1 βi = Do vậy, theo giả thiết quy nạp ta k k−1 λi f (xi ) ≥ (1 − λk ) i=1 βi f (xi ) + λk f (xk ) i=1 k−1 ≥ f [λk xk + (1 − λk ) βi xi ] i=1 k ≥f λi x i i=1 Định lý chứng minh ✷ • Một số hệ 1) Nếu hàm số y = f (x) lồi tập lồi M với x1 , x2 , , xn ∈ M ta có bất đẳng thức f x1 + x2 + + xn f (x1 ) + f (x2 ) + + f (xn ) ≤ , (n ≥ 2) n n (3) 2) Nếu hàm số y = f (x) lồi M lồi với x1 , x2 , , xn ∈ M ta có bất đẳng thức f m1 f (x1 ) + m2 f (x2 ) + + mn f (xn ) m1 x1 + m2 x2 + + mn xn ≤ (4) m1 + m2 + + mn m1 + m2 + + mn Thật vậy, áp dụng bất đẳng thức Jensen (2) lấy với λ1 = λ2 = = λn = n1 , ta có bất đẳng thức (3), lấy với λi = mi m1 +m2 + +mn , mi > 0, i = 1, 2, , n, ta có bất đẳng thức (4) 3) Hàm f liên tục tập M hàm lồi f x+y ≤ (f (x) + f (y)) 2 (5) Chứng minh Điều kiện cần rõ ràng từ bất đẳng thức lồi (1) với λ thuộc [0, 1] Điều kiện đủ Ta cần chứng minh có (5) hàm liên tục f lồi Từ (5) ta có f x(1) + x(2) + x(3) + x(4) ≤ f (x(1) ) + f (x(2) ) + f (x(3) ) + f (x(4) ) 4 Bằng quy nạp hoàn toàn với m = 2k , k số tự nhiên đó, ta nhận bất đẳng thức f m m x (i) i=1 ≤ m m f (x(i) ) (∗) i=1 Ta chứng minh quy nạp bất đẳng thức (*) với số tự nhiên m Rõ ràng với m = 2k đủ lớn (*) Gải sử với m = N bất đẳng thức đúng, tức f N N x i=1 (i) ≤ N N f (x(i) ) i=1 10 Ta chứng minh bất đẳng thức với m = N − Đặt (x(1) + x(2) + + x(N −1) ), N −1 x(N ) = ta có x(N ) = (1) (x + x(2) + + x(N ) ), N N f (x (N ) )=f x ≤ N (i) i=1 N N −1 f (x ) = N (i) i=1 f (x(i) ) + i=1 f (x(N ) ) N Từ ta N −1 f x i=1 (i) ≤ N −1 N −1 f (x(i) ) i=1 Như (*) với m số tự nhiên Để chứng minh f lồi, ta phải chứng minh f thỏa mãn bất đẳng thức lồi (1) Lấy x, y ∈ M k, m số tự nhiên, k < m Từ (*) ta có f kx + (m − k)y m Đặt λ = k m, =f k m−k x+ y m m ≤ kf (x) + (m − k)f (y) m ta có λ số hữu tỉ thỏa mãn < λ < f (λx + (1 − λ)y) ≤ λf (x) + (1 − λ)f (y) Vì f hàm liên tục, bất đẳng thức lồi với giá trị thực λ thuộc đoạn [0, 1] 4) Cho M tập hợp lồi, f (x) hàm liên tục lồi xác định khơng âm M , f (x) hàm lồi M Thật vậy, M lồi nên x, y thuộc M x+y thuộc M Lại f lồi nên theo (5) f x+y ≤ [f (x) + f (y)] 2 11 Do f hàm không âm nên f2 x+y ≤ [f (x) + f (y)]2 Mặt khác ta có 1 1 [f (x) + f (y)]2 − f (x) − f (y) = − [f (x) + f (y)] 2 Nghĩa f (x) hàm lồi M 5) Tổng hữu hạn hàm lồi xác định tập lồi M hàm lồi Chú ý rằng, hiệu hai hàm lồi nói chung hàm lồi 6) Cho f (x) hàm lồi tập lồi M số thực α cố định Khi tập Mα = {x ∈ Rn : f (x) ≤ α} tập lồi Tổng quát ta có: Cho fi , i = 1, 2, , k hàm lồi xác định tập lồi M Khi tập hợp Mα = {x ∈ M : fi (x) ≤ αi , i = 1, 2, , k, αi ∈ R} tập lồi 1.1.3 Ví dụ a) Chứng minh bất đẳng thức số học • Bất đẳng thức Cơsi √ a1 + a2 + + an ≥ n a1 a2 an n Giải: Ta có hàm −lnx hàm lồi, áp dụng bất đẳng thức Jensen dạng đơn giản f x1 + x2 + + xn f (x1 ) + f (x2 ) + + f (xn ) ≤ n n 12 Ta có −ln a1 + a2 + + an ln(a1 a2 an ) ≥ n n √ a1 + a2 + + an ⇔ ≥ n a1 a2 an n • Bất đẳng thức Bunhiacôpxki (a1 b1 + a2 b2 + + an bn )2 ≤ (a21 + a22 + + a2n )(b21 + b22 + + b2n ) a1 , a2 , , an ; b1 , b2 , , bn 2n số tuỳ ý Giải: Xét hàm lồi f (x) = x2 với x ∈ R áp dụng bất đẳng thức Jensen dạng tổng quát ta có m1 x1 + m2 x2 + + mn xn m1 x21 + m2 x22 + + mn x2n ≤ m1 + m2 + + mn m1 + m2 + + mn ⇔ (m1 x1 +m2 x2 + +mn xn )2 ≤ (m1 +m2 + +mn )(m1 x21 +m2 x22 + +mn x2n ) Đặt mi = b2i xi = a1 b21 b1 + a2 b22 b2 + + bi an b2n bn ta có ≤ (b21 + b22 + + 2 2 a1 a2 an b1 + b2 + + bn b1 b2 bn + a2n )(b21 + b22 + + b2n ) b2n ) ⇔ (a1 b1 + a2 b2 + + an bn ) ≤ (a21 + a22 + • Bất đẳng thức Mincơpxki √ n a1 a2 an + n b1 b2 bn ≤ n (a1 + b1 )(a2 + b2 ) + + (an + bn ) a1 , a2 , , an ; b1 , b2 , , bn 2n số dương Giải: Xét hàm số f (x) = ln(1 + ex ) Khi f (x) = ex ex , f ”(x) = > 0, ∀x ∈ R + ex (1 + ex )2 Vì f (x) hàm lồi R áp dụng bất đẳng thức Jensen dạng đơn giản f x1 + x2 + + xn f (x1 ) + f (x2 ) + + f (xn ) = n n 20 2.2 Tính chất Người ta chứng minh rằng: Nếu hàm số f : I → (0, +∞) thoả mãn x2 f ”(x) + xf (x) ≥ (2.4) hàm GA-lồi Trong trường hợp (2.4) khơng thoả mãn Tiếp theo nghiên cứu phần câu trả lời cho vấn đề đặt Giả thiết f : (0, +∞) → (0, +∞) hàm có vi phân bậc hai thỏa mãn f ”(x) ≥ miền xác định Đặt g : (0, +∞) → (0, +∞), với g(x) = x2 f ”(x) + xf (x) Bất đẳng thức (2.4) không thoả mãn, nghĩa tồn số thực r : < r < 1, g(r) = cho g < (0, r) g ≥ (r, ∞) Xét hàm h : (0, +∞)n → (0, +∞) xác định n f (xk ) − nf (1), h(x1 , x2 , , xn ) = k=1 với x1 , x2 , , xn > thoả mãn n xk = (2.5) k=1 Ta giả thiết toạ độ điểm tới hạn h thoả mãn (2.5) lấy giá trị khác nhau, nghĩa tồn a = b cho x1 , x2 , , xn = a, b điểm tới hạn hợp thành (x1 , x2 , , xn ) h Định lý 2.1 Nếu tất điều kiện nêu thoả mãn k = 1, 2, , n, có lim h(x1 , x2 , , xn ) ≥ 0, (2.6) xk →0 h(x1 , x2 , , xn ) ≥ 0, với x1 , x2 , , xn ≥ 0, thoả mãn n k=1 xk = Chứng minh Trước hết ta ý h hàm liên tục, xác định tập bị chặn (từ điều kiện), nên tồn giá trị m > −∞ cho h ≥ m, 21 cho x1 , , xn > 0, với n k=1 xk = Theo yêu cầu định lý, cần m ≥ Ngồi ra, từ (2.6) tất yếu suy tính bị chặn miền xác định, nghĩa có giới hạn xk → 0, với k = 1, , n Đặt n xk = K = (x1 , x2 , , xn ) : x1 , x2 , , xn > 0, k=1 Việc lại chứng minh ta cần thiết lập xác định phần K Chúng ta xét điểm cực trị h, tìm từ điểm tới hạn Theo giả thiết, toạ độ chúng lấy giá trị khác Điều suy ∂h = 0, i = 1, , n ⇒ {x01 , x02 , , x0n } = {a, b} ∂xi tất điểm tới hạn Vì tính đối xứng, giả thiết x01 ≤ x02 ≤ ≤ x0n a ≤ b Do tồn q, với ≤ q ≤ n, cho x01 = x02 = = x0q−1 = a x0q = x0q+1 = = x0n = b (khi q = 1, quy ước x00 = 0) Để ý theo (2.5) suy b ≥ Chú ý q = khơng cần chứng minh trường hợp kết luận điều kiện (2.4) cho ta điểm cực tiểu (hoặc suy trực tiếp từ (2.5)) Xét tiếp điều kiện h1 (a, b) = (q − 1)f (a) + (n − q + 1)f (b) − nf (1) (2.7) Chú ý theo (2.5) có aq−1 bn−q+1 = (2.8) Chúng ta h1 (a, b) ≥ 0, với a, b > thoả mãn (2.8) Theo (2.8) điều tương đương với h1 (b) = (q − 1)f b n+1−q 1−q + (n − q + 1)f (b) − nf (1) ≥ (2.9) 22 Tính tốn cho thấy h1 (b) = h1 (b) = h1 b n+1−q 1−q n b 1−q Bởi b ≥ f hàm tăng nên b = (2.9) trở thành đẳng thức Ngồi ra, h”1 (1) = n q−1 (f ”(1) + f (1)) ≥ điều kiện (2.4) sử dụng từ g x = r < Điều b0 = điểm cực tiểu h1 đẳng thức (2.9) điểm Điều cho thấy với điểm khác b ≥ Cuối cùng, điều cho ta đắn điểm cực tiểu f với tất điểm miền xác định K Chúng ta trực tiếp thử lại điểm biên (điểm giới hạn) K cho thấy kết ✷ tương tự Định lý chứng minh xong 2.3 Một số áp dụng Trong tài liệu tham khảo [5], tác giả chứng minh bất đẳng thức n i=1 xn−1 (1 + λ) n−1 i xi + λΠk=i xk n−1 ≥ n, (2.10) với n ≥ 1, xi > 0, i = 1, 2, , n λ ≥ nn−1 − Bây ta đặt yi = Πk=i xk , i = 1, 2, , n, xn−1 i ta có định lý sau đây: Định lý 2.2 Ta có n √ n−1 i=1 ≥ + λyi √ n−1 n 1+λ (2.11) với n ≥ yi > 0, i = 1, 2, , n, Πni=1 yi = 1, λ ≥ nn−1 − Chứng minh Trong trường hợp n = 1, bất đẳng thức (2.11) Ta xét với n > 23 Với n > x, λ > 0, giả sử f (x) = (1 + λx)−1/(n−1) Khi f (x) = −λ(n − 1)−1 (1 + λx)−n/(n−1) < 0, f ”(x) = λ2 n(n − 1)−2 (1 + λx)−(2n−1)/(n−1) > 0, với x > với λ > 0, n > Điều cho thấy f hàm giảm lồi theo biến x Với λ > 0, ta có (1 + λx)(1−2n)/(n−1) x f ”(x) + xf (x) = λx (1 + λx − n) λ2 Khi x2 f ”(x) + xf (x) ≥ + λx − n > Điều xảy x > n−1 λ Do khoảng J = giả thiết cho ta có n−1 λ ,∞ hàm f hàm GA-lồi Từ n yi = (2.12) i=1 Vì (2.11) trở thành n f (yi ) − nf (1) ≥ 0, hh(y1 , y2 , , yn ) = (2.13) k=1 với yi > 0, i = 1, 2, , n thoả mãn (2.13) λ ≥ nn−1 − Từ ta nhận thấy điểm tới hạn hàm hh thoả mãn điều kiện (2.12) phải thoả mãn đẳng thức d(y1 ) = d(y2 ) = = d(yn ), d(y) = y (1+λy)n/(n−1) Lúc d hàm đơn điều ngặt J R − J, nên ta suy điểm tới hạn hàm hh (2.13) đạt giá trị a, b (a ≤ b) Ngoài (2.12) cho ta b ≥ Tới đây, thấy có ngoại lệ điều kiện (2.6) Tất giả thiết định lý 2.1 thoả mãn trường hợp 24 xét, chúng kết thúc với tất điểm miền xác định Bây xét tới trường hợp miền xác định hàm hh (2.13) a → 0, (hoặc tương đương b → 0) Bởi lim f (a) = 1, lim f (b) = 0, a→0 b→∞ nên có q − ≥ nf (1) = n(1 + λx)−1/(n−1) Điều hiến nhiên với điều kiện λ ≥ nn−1 − Đẳng thức xảy với q = λ = nn−1 − Từ suy giả thiết (2.2) cố định trường hợp Bởi bất đẳng thức (2.13) thoả mãn với tất điểm tới hạn yi > 0, i = 1, 2, , n λ ≥ nn−1 − Sử dụng định lý ✷ 2.1, điều phải chứng minh Định lý 2.3 Với α, β > 0, n ≥ cho β ≥ (nn−1 − 1)α, có bất đẳng thức n i=1 xn−1 i n−1 αxi + βΠk=i xk n−1 −1 ≥ n(α + β) n−1 (2.14) Chứng minh Việc chứng minh định lý dễ dàng suy từ định lý 2.1, thông qua lý luận làm với định lý 2.2 ✷ Từ định lý 2.1, ta có định lý 2.4 sau đây: Định lý 2.4 Với n ≥ 1, p ≥ 1, yi > 0, i = 1, 2, , n cho Πni=1 yi = λ ≥ np − 1, có bất đẳng thức n (1 + λyi )−1/p ≥ n(α + β)−1/p (2.15) i=1 Chứng minh Việc chứng minh định lý 2.4 thực định lý 2.2 cách thay n − p 25 Từ định lý 2.2 định lý 2.4, có định lý mà công thức đối ngẫu với công thức (2.10) sau: Định lý 2.5 Với n ≥ 1, p ≥ 1, xi > 0, i = 1, 2, , n; α, β > 0; λ ≥ np − 1, có bất đẳng thức n i=1 Πk=i xk (α + λβ) Πk=i xk + λxn−1 i −1/p ≥ n (2.16) Chứng minh Bất đẳng thức (2.16) suy từ (2.14) (2.15) ✷ thay xi 1/xi , i = 1, 2, , n Hệ Nếu α, β > thoả mãn β ≥ (nn−1 − 1)α n −1 −1 (α + βxji ) n−1 ≥ n(α + β) n−1 , (2.17) j=1 với n ≥ 1, xi > 0, i = 1, 2, , n, cho Πni=1 xi = Chứng minh Từ (2.16) thay p = n − 1, nhân vế bất đẳng thức với (α + β)−1/(n−1) , thực phép biến đổi ta điều phải ✷ chứng minh 2.4 Một số ví dụ Trong mục này, xét số toán đặc biệt sau: 2.4.1 Bài toán Chứng minh √ a b c +√ +√ ≥ 1, a2 + 8bc b2 + 8ca c2 + 8ab với số thực dương a, b, c Chứng minh Ta chọn số dương r cho √ a a2r ≥ 2r a + 2(bc)r a2 + 8bc Bất đẳng thức tương đương với a2 (a2r + 2br cr )2 ≥ a4r (a2 + 8bc) 26 Điều xảy b2r c2r + a2r br cr ≥ 2a4r−2 bc Theo bất đẳng thức Cơsi b2r c2r + a2r br cr ≥ 2ar b3r/2 c3r/2 Ta chí cần chọn r cho    4r − = r   3r/2 = Từ suy r = 2/3 Khi theo bất đẳng thức AM-GM ta √ a b c +√ +√ ≥ a2 + 8bc b2 + 8ca c2 + 8ab a2r b2r c2r ≥ 2r + + a + 2br cr b2r + 2ar cr c2r + 2ar br a2r b2r c2r ≥ 2r + + = a + b2r + c2r a2r + b2r + c2r a2r + b2r + c2r Đó điều phải chứng minh ✷ Chú ý 1) Từ bất đẳng thức nêu trên, ta có đẳng thức xảy a = b = c 2) Phát biểu tổng quát cho toán là: Đối với a, b, c > λ ≥ ta có bất đẳng thức b c a +√ +√ ≥√ 1+λ a2 + λbc b2 + λca c2 + λab 2.4.2 Bài toán Chứng minh a, b, c số thực dương √ bất đẳng thức sau ln ln 1 + + ≥ a3 (1 + b3 ) b3 (1 + c3 ) c3 (1 + a3 ) abc(1 + abc) 27 Chứng minh Ta có a3 b c + + a3 b + a3 + a3 b c + = a3 (b3 + 1) a3 (b3 + 1) a3 b3 (1 + c3 ) + a3 + = a3 (b3 + 1) b3 (1 + c3 ) a3 + = + + b3 ) a (1 + b3 ) Vậy theo bất đẳng thức AM-GM + (a3 b3 c3 + 1) = cyc a3 (1 1 + + 3 + b ) b (1 + c ) c (1 + a3 ) a3 + + a3 (1 + b3 ) cyc = b3 (1 + c3 ) ≥ + 3abc, + b3 abc cyc cyc a3 + b3 + c3 + a3 + = + + a3 (1 + b3 ) a3 (1 + b3 ) b3 (1 + c3 ) c3 (1 + a3 ) b3 (1 + c3 ) b3 (1 + c3 ) c3 (1 + a3 ) a3 (1 + b3 ) = + + + b3 + b3 + c3 + a3 Mặt khác ta có a3 b c + a3 b c + ≤ abc(1 + abc) abc 1 = a2 b c + ≤ + abc − abc abc + a2 b2 c2 − abc = abc Từ ta có 1 3(1 + a2 b2 c2 − abc) + + ≥ a3 (1 + b3 ) b3 (1 + c3 ) c3 (1 + a3 ) a3 b c + 3(1 + a2 b2 c2 − abc) ≥ abc(1 + abc) + a2 b2 c2 − abc = abc(1 + abc) 28 ✷ Đó điều phải chứng minh Bất đẳng thức nêu toán 2, xảy đẳng thức a = b = c = 2.4.3 Bài toán Giả sử a1 , a2 , , an số dương, với a1 a2 an = Chứng minh 1 + (n2 − 1)a1 + + (n2 − 1)a2 + + 1 + (n2 − 1)an > Chứng minh Đặt xi = 1 + (n2 − 1)ai , ∀i = 1, n; p = x1 x2 xn Ta chứng minh phản chứng Giả sử x1 + x2 + + xn < Khi 1 − x2i = = x2i − 1) n2 − (n2 − 1)x2i Từ suy n (1 − x2i ) = (n2 − 1)n p2 (∗) i=1 Vì n i=1 xi < 1, nên theo bất đẳng thức Côsi ta có n − xj > −xj + xi ≥ (n − 1) n−1 i=1 n + xj ≥ xj + p , xj √ xi ≥ (n + 1) n+1 pxj i=1 Lấy j = 1, 2, , n, nhân tất bất đẳng thức ta n (1 − x2i ) > (n2 − 1)n p2 i=1 Điều mâu thuẫn với (*) Vậy ta có điều phải chứng minh ✷ Bất đẳng thức nêu toán trở thành đẳng thức a1 = a2 = = an = 29 2.4.4 Bài toán Cho a1 , a2 , , an số thực dương thoả mãn a1 a2 an = cho trước n(n ∈ N, n > 2) Chứng minh a1 + a2 + an + + + + ≥ (a1 + 1)2 (a2 + 1)2 (an + 1)2 Chứng minh Ta xét với n = Lúc cách chọn số thực dương a, b, c cho a b c = = = k ≥ 1; abc = a1 b1 c1 Khi ta thấy a1 + a2 + a3 + a+3 b+3 c+3 + + ≥ + + 2 2 (a1 + 1) (a2 + 1) (a3 + 1) (a + 1) (b + 1) (c + 1)2 Đặt a1 = 2 , b1 = , c1 = 1+a 1+b 1+c Ta phải chứng minh a1 + b1 + c1 + a21 + b21 + c21 ≥ Chú ý với điều kiện abc = 1, cho ta 1 − a1 1 − b1 1 abc − = = c1 8 Điều xảy (2 − a1 )(2 − b1 )(2 − c1 ) = a1 b1 c1 Đặt x = a1 − 1, y = b1 − 1, z = c1 − 1, x, y, z ≥ −1, nên (x + 1)(y + 1)(z + 1) = (1 − x)(1 − y)(1 − z) Từ suy x + y + z + xyz = Do ≥ x, y, z ≥ −1 nên x2 + y + z ≥ 3(xyz)2/3 ≥ 3xyz 30 Vậy (a1 − 1)(a2 + 2) = a1 + b1 + c1 + a21 + b21 + c21 − ≥ x(x + 3) = sym x,y,z Như vậy, ta chứng minh bất đẳng thức toán với n = Trong trường hợp tổng quát, ta cần chọn số thực dương a1 , a2 , a3 cho a1 a2 a3 < 1, lúc ta dẫn trường hợp xét với n = nhờ bất đẳng thức sau n i=1 + a1 + a2 + a3 + ≥ + + (ai + 1)2 (a1 + 1)2 (a2 + 1)2 (a3 + 1)2 ✷ Đó điều phải chứng minh Chú ý bất đẳng thức toán 4, trở thành đẳng thức a1 = 0, a2 = = an = +∞ 2.4.5 Bài toán Chứng minh a, b, c số thực dương có tích a+b + a+1 b+c + b+1 c+a ≥ c+1 Chứng minh Sử dụng trực tiếp bất đẳng thức AM-GM cho số hạng toán, ta chứng minh (a + b)(b + c)(c + a) ≥ (a + 1)(b + 1)(c + 1) Thật vậy, với điều kiện abc = 1, bất đẳng thức nêu tương đương với ab(a + b) + bc(b + c) + ca(c + a) ≥ a + b + c + ab + bc + ca Sử dụng bất đẳng thức AM-GM với nhóm số, ta có a2 b + a2 b + a2 c + a2 c + bc ≥ 5a, 31 b2 a + b2 a + b2 c + b2 c + ac ≥ 5b, c2 b + c2 b + c2 a + c2 a + ab ≥ 5c Cộng bất đẳng thức (với ký hiệu VT kết vế trái nêu trên) 2.V T ≥ 5(a + b + c) − (ab + bc + ca) (∗) Tương tự, áp dụng bất đẳng thức AM-GM cho b2 c + b2 c + c2 b + c2 b + a ≥ 5bc, b2 a + b2 a + a2 b + a2 b + c ≥ 5ab, a2 c + a2 c + c2 a + c2 a + b ≥ 5ca, ta 2.V T ≥ 5(ab + bc + ca) − (a + b + c) Cộng (*) với (**) ta có điều phải chứng minh (∗∗) ✷ 2.4.6 Bài toán Chứng minh a, b, c, d số thực dương có tích 1 1 + + + ≥ (1 + a)2 (1 + b)2 (1 + c)2 (1 + d)2 Chứng minh Ta thấy với x, y hai số thực dương tuỳ ý ln có 1 + ≥ (1 + x)2 (1 + y)2 (1 + xy) Thật vậy, khai triển biến đổi tương đương ta (2 + 2x + 2y + x2 + y )(1 + xy) ≥ (1 + 2x + x2 )(1 + 2y + y ) Điều xảy + 2(x + y) + x2 + y + 2xy + 2xy(x + y) + xy(x2 + y ) ≥ 32 ≥ + 2y + y + 2x + 4xy + y + x2 + 2xy(x + y) + x2 y Bất đẳng thức nêu tương đương với + xy(x2 + y ) ≥ 2xy + x2 y ⇔ (xy + 1)(x − y)2 ≥ Đặt m = ab, n = cd mn = Do 1 m+n+2 + = = 1 + m + n (m + 1)(n + 1) Sử dụng kết ta có 1 1 1 + + + ≥ + = (1 + a)2 (1 + b)2 (1 + c)2 (1 + d)2 1+m 1+n Đó điều phải chứng minh ✷ Chú ý bất đẳng thức toán trở thành đẳng thức a = b = c = d = 33 Kết luận Kết thực đề tài khoá luận bao gồm: Trình bày khái niệm kiến thức sở hàm lồi, hàm lồi biến thông qua khái niệm, tính chất số ví dụ áp dụng Trình bày khái niệm hàm AG-lồi, GG-lồi, GA-lồi Nghiên cứu số tính chất, thông qua bất đẳng thức hàm nêu Đưa toán chứng minh cách sử dụng bất đẳng thức quan nêu khố luận Do thời gian trình độ có hạn nên số vấn đề cần tiếp tục nghiên cứu bào gồm: - Nghiên cứu tốn hình học việc sử dụng bất đẳng thức nêu - Khai thác nghiên cứu mở rộng số bất đẳng thức khác có ý nghĩa khoa học 34 Tài liệu tham khảo [1] Phạm Kim Hùng, Sáng tạo bất đẳng thức, NXB Hà Nội, Hà Nội, 2007 [2] Đỗ Văn Lưu - Phan Huy Khải, Giải tích lồi, NXB Khoa học Kỹ thuật, Hà Nội, 2000 [3] Trần Xuân Sinh, Giải tích lồi, Đại học Vinh, 2006 [4] Jan Van Tiel, Convex Analysis, Royal Netherianas Meteorological Institute, 1984 [5] C.P Nicullescu, Convexity According to the Geometric mean, J Mathmatical Inequaties and Applications, Vol.3, N0 (2000), 155-167 [6] R.A Satnoianu, Improved GA-convexity inequalities, Journal of Inequalities in Pure and Applied Mathmatics, Vol 3, Tssue 5, Article 82, 2002, ISSN (Electronic): 1443-5756 ... tài Các bất đẳng thức AG- lồi, GG -lồi, GA- lồi ứng dụng Nội dung luận văn bao gồm hai chương: Chương 1: Bất đẳng thức lồi Trong chương này, chúng tơi trình bày kiến thức sở có liên quan đến bất đẳng. .. tính chất số ví dụ áp dụng Trình bày khái niệm hàm AG- lồi, GG -lồi, GA- lồi Nghiên cứu số tính chất, thơng qua bất đẳng thức hàm nêu Đưa toán chứng minh cách sử dụng bất đẳng thức quan nêu khoá luận... kiến thức sở có liên quan đến bất đẳng thức lồi, là: Khái niệm hàm lồi không gian Rn , Bất đẳng thức Jensen, Hàm lồi biến số số ứng dụng Chương 2: Bất đẳng thức số học hình học Đây nội dung khố

Ngày đăng: 21/10/2021, 23:11

HÌNH ẢNH LIÊN QUAN

b) Chứng minh các bất đẳng thức hình học - Các bất đẳng thức ag lồi, gg lồi, ga lồi và ứng dụng
b Chứng minh các bất đẳng thức hình học (Trang 11)

TỪ KHÓA LIÊN QUAN

TRÍCH ĐOẠN

TÀI LIỆU CÙNG NGƯỜI DÙNG

TÀI LIỆU LIÊN QUAN

w